Mathcenter Forum

Mathcenter Forum (https://www.mathcenter.net/forum/index.php)
-   ข้อสอบโอลิมปิก (https://www.mathcenter.net/forum/forumdisplay.php?f=28)
-   -   TMO11 (https://www.mathcenter.net/forum/showthread.php?t=21032)

Thgx0312555 15 พฤษภาคม 2014 20:46

TMO11
 
รบกวนคนที่ไปสอบ tmo มา โพสต์ข้อสอบด้วยครับ

polsk133 16 พฤษภาคม 2014 16:32

5. จงหาจำนวนจริง k ที่มากที่สุดที่ทำให้อสมการ $$(k+\frac{a}{b})(k+\frac{b}{c})(k+\frac{c}{a}) \leqslant (\frac{a}{b}+\frac{b}{c}+\frac{c}{a})(\frac{b}{a}+\frac{c}{b}+\frac{a}{c})$$ เป็นจริงทุกจำนวนจริงบวก $a,b,c$

6. จงหาจำนวนเฉพาะ p (บวก) ที่ทำให้ $2p^2-3p-1$ เป็นกำลังสามของจำนวนเต็มบวก

passer-by 16 พฤษภาคม 2014 18:23

อ้างอิง:

ข้อความเดิมเขียนโดยคุณ polsk133 (ข้อความที่ 170489)
5. จงหาจำนวนจริง k ที่มากที่สุดที่ทำให้อสมการ $$(k+\frac{a}{b})(k+\frac{b}{c})(k+\frac{c}{a}) \leqslant (\frac{a}{b}+\frac{b}{c}+\frac{c}{a})(\frac{b}{a}+\frac{c}{b}+\frac{a}{c})$$ เป็นจริงทุกจำนวนจริงบวก $a,b,c$

(Credit : ครึ่งบน inspired by P' noonuii)
เปลี่ยนตัวแปรเป็น x,y,z โดย xyz =1

ดังนั้นอสมการที่จะพิสูจน์สมมูลกับ $ (k+x)(k+y)(k+z) \leq (x+y+z)(xy+yz+zx) \Rightarrow (x+y+z -k)(xy+yz+zx -k^2) \geq 1+2k^3$

อันดับแรก จะพิสูจน์ $k = \sqrt[3]{9} -1 $ เป็นไปได้ (สังเกตว่า $(k+1)^3 =9$)

By AM-GM $ x+y+z \geq 3 >k \,\, , xy+yz+zx \geq 3 >k^2$ ดังนั้น $(x+y+z -k)(xy+yz+zx -k^2) \geq (3-k)(3-k^2) $

และ $ (3-k)(3-k^2) = 1+2k^3$ เพราะสมมูลกับ $(k+1)^3 =9$

----------------------------------------------

ต่อไปจะ prove such k maximum

take y = $\frac{1}{x}$ และ z=1

ถ้า k สอดคล้องกับอสมการ $ (k+x)(k+ \frac{1}{x})(k+1) \leq (x+\frac{1}{x}+1)^2$

Take limit x เข้าใกล้ 1 ดังนั้น $ (k+1)^3 \leq 9 $

p.s. มีคนถามผมว่า ไม่ take limit ได้มั้ย คำตอบคือได้ เช่น แทน 1,1,1 เพียงแต่ ผมตอบจากความเคยชินแวบแรก เวลาเห็นโจทย์สไตล์นี้ เพราะ บางข้อ มัน แทน 1,1,1 ไม่ได้ อาจต้องใช้ตัวอย่างละเอียดแล้ว take limit เช่น หาจำนวนจริงบวก c น้อยสุดที่ ทำให้ $$ \sum_{k=1}^n \frac{k}{\frac{1}{a_1} +\frac{1}{a_2}+...+\frac{1}{a_k}} < c \sum_{k=1}^n a_k \,\,\, ,\forall a_i>0 $$

Poogunexe 16 พฤษภาคม 2014 19:47

คราวนี้ดีนะครับ มีเฉลยแจกให้ด้วยหลังสอบเสร็จ

Poogunexe 16 พฤษภาคม 2014 19:55

2. จงหาฟังก์ชัน $ f:\mathbb{R} \rightarrow \mathbb{R} $ ทั้งหมดที่สอดคล้องกับเงื่อนไข
$ f(xy-1)+f(x)f(y)=2xy-1 $
สำหรับทุกจำนวนจริง $x$ และ $y$

Beatmania 16 พฤษภาคม 2014 20:08

ลงฉบับเต็มครับ :D

1. ให้ $ABC$ เป็นสามเหลี่ยมหน้าจั่วที่มี $BAC=100$ ต่อด้าน $AB$ ออกไปทาง $B$ ถึง $D,E$ โดยที่ $BC=AD=BE$

จงแสดงว่า $(BC)(DE)=(BD)(CE)$

2. จงหาฟังก์ชั่น $f:\mathbb{R} \rightarrow \mathbb{R}$ ทั้งหมดที่สอดคล้องกับสมการ

$$f(xy-1)+f(x)f(y)=2xy-1$$

ทุกจำนวนจริง $x,y$

3. ให้ $M,N$ เป็นจำนวนเต็มบวก นายพิสุทธ์เดินจากจุด $(0,N)$ ไปยังจุด $(M,0)$ โดยที่

i.) แต่ละก้าวของเขายาว $1$ หน่วย ไปในทิศขนานแกน $X$ หรือแกน $Y$

ii.) ทุก ๆ จุด $(x,y)$ ที่เขาเดินผ่านมีค่า $x\geq 0$ และ $y\geq 0$

iii.) ในแต่ละก้าว เขาจะวัดระยะทางจากตัวเขาไปยังแกนที่เขาเดินขนาน

ถ้าเขาเดินแล้วไกลจากจุดกำเนิดมากขึ้น เขาจะบันทึกระยะทางดังกล่าวเป็นจำนวนบวก

ในทางกลับกัน ถ้าเขาเดินแล้วใกล้จากจุดกำเนิดมากขึ้น เขาจะบันทึกระยะทางดังกล่าวเป็นจำนวนลบ

จงพิสูจน์ว่า หลังจากการเดินเสร็จสิ้นแล้ว ผลรวมระยะทางที่เขาบันทึกได้จะเป็น 0

4. จงหาพหุนาม $P(x)$ ที่มีสัมประสิทธ์เป็นจำนวนเต็มที่

$$P(n)|2557^n+213\times 2014$$

ทุกๆ จำนวนเต็มบวก $n$

Beatmania 16 พฤษภาคม 2014 20:21

ฉบับเต็ม วันที่สอง

5. จงหาจำนวนจริง $k$ ที่มากที่สุดที่ทำให้

$$(k+\frac{a}{b})(k+\frac{b}{c})(k+\frac{c}{a})\leq (\frac{a}{b}+\frac{b}{c}+\frac{c}{a})(\frac{b}{a}+\frac{c}{b}+\frac{a}{c})$$

ทุกๆ จำนวนจริงบวก $a,b,c$

6. จงหาจำนวนเฉพาะ $p$ ที่ทำให้ $2p^2-3p-1$ เป็นกำลังสามสมบูรณ์ของจำนวนเต็มบวก

7.ให้ $ABCD$ เป็นสี่เหลี่ยมนูนโดยที่มีด้าน $AB,CD$ สั้นที่สุดและยาวที่สุดตามลำดับและ $AB\neq CD$

จงแสดงว่ามีจุด $E$ อยู่ระหว่าง $C,D$ โดยที่

"สำหรับจุด $P$ ใดๆ $(P\neq E)$ บนด้าน $CD$ ที่อยู่ระหว่าง $C,D$ ความยาวของ $O_1O_2$ เป็นค่าคงที่"

เมื่อ $O_1,O_2$ เป็น $Circumcenter$ ของ $APD,BPE$ ตามลำดับ

8. ให้ $n$ เป็นจำนวนเต็มบวก ต้องการสร้างบัตรชุดหนึ่งที่มีเงื่อนไขว่า

i.) ตัวเลขที่ปรากฎบนบัตรอยู่ในรูป $m!$ เมื่อ $m$ เป็นจำนวนเต็มบวก

ii.) สำหรับจำนวนนับ $t$ ใดๆ ที่ $t\leq n!$ เราสามารถเลือกบัตรจำนวนหนึ่งจากบัตรชุดนี้โดยให้ผลรวมของตัวเลขบนบัตรมีค่าเท่ากับ $t$

จงหาว่าการสร้างบัตรชุดนี้จะต้องใช้บัตรอย่างน้อยที่สุดกี่ใบ

nooonuii 16 พฤษภาคม 2014 20:51

อ้างอิง:

ข้อความเดิมเขียนโดยคุณ passer-by (ข้อความที่ 170490)
(Credit : ครึ่งบน inspired by P' noonuii)
เปลี่ยนตัวแปรเป็น x,y,z โดย xyz =1

ดังนั้นอสมการที่จะพิสูจน์สมมูลกับ $ (k+x)(k+y)(k+z) \leq (x+y+z)(xy+yz+zx) \Rightarrow (x+y+z -k)(xy+yz+zx -k^2) \geq 1+2k^3$

อันดับแรก จะพิสูจน์ $k = \sqrt[3]{9} -1 $ เป็นไปได้ (สังเกตว่า $(k+1)^3 =9$)

By AM-GM $ x+y+z \geq 3 >k \,\, , xy+yz+zx \geq 3 >k^2$ ดังนั้น $(x+y+z -k)(xy+yz+zx -k^2) \geq (3-k)(3-k^2) $

และ $ (3-k)(3-k^2) = 1+2k^3$ เพราะสมมูลกับ $(k+1)^3 =9$

วิธีนี้จริงๆแล้วมาจากผู้เข้าสอบบางคนครับ ผมว่าสวยดีแต่เสียดายที่ไม่มีใครทำวิธีนี้ได้เต็มไม่งั้นคงได้ best solution

ตั้งใจให้เป็นโจทย์ง่ายนะเนี่ย แต่กลายเป็นโจทย์ที่ยากมากๆไปได้ยังไงก็ไม่รู้ :sweat:

Beatmania 16 พฤษภาคม 2014 20:55

#8 ผมคิดว่ามันยากตรงที่ตรรกศาสตร์และการเขียนนะครับ เจอโจทย์แบบนี้ผมเขียนไม่เคยถูกเลย TT

แถมปีนี้ผมว่ามีข้อที่เขียนยากอยู่หมายข้อ เช่น 3,5,8 :sweat:

passer-by 16 พฤษภาคม 2014 21:37

อ้างอิง:

ข้อความเดิมเขียนโดยคุณ Poogunexe (ข้อความที่ 170492)
2. จงหาฟังก์ชัน $ f:\mathbb{R} \rightarrow \mathbb{R} $ ทั้งหมดที่สอดคล้องกับเงื่อนไข
$ f(xy-1)+f(x)f(y)=2xy-1 $
สำหรับทุกจำนวนจริง $x$ และ $y$

Take y =0 , จะได้ $ f(-1) +f(x)f(0) = -1 $

แต่ constant function ไม่ใช่คำตอบ ดังนั้น f(0) = 0

Take $ y= \frac{1}{x}$ ดังนั้น $ f(x) f(1/x) = 1- f(0) = 1 \,\, ,\forall x \neq 0 .......(1)$

จาก (1) implies f(1) = 1 or -1

Take y= 1 ใน original จะได้ $f(x-1) + f(x)f(1) = 2x-1 ......(2)$

Take $ y = \frac{1}{x-1} $ จะได้ $ f(\frac{1}{x-1})(1+f(x)) = \frac{x+1}{x-1} \,\, \forall x \neq 1 .......(3)$

Using (1) ใน (3) ดังนั้น $\frac{1}{f(x-1)}(1+f(x)) = \frac{x+1}{x-1} ......(4) $

Using (2) ใน (4) จะได้ $\frac{1}{2x-1-f(x)f(1)}(1+f(x)) = \frac{x+1}{x-1} $

ตอนนี้มีแต่ f(x) อย่างเดียวแล้ว :happy:
แทน f(1) แต่ละกรณีลงไป แล้ว simplify บรรทัดก่อนหน้า จะได้ $ f(x) = x$ และ $ f(x) = -x^2$

แทนในโจทย์แล้วจริงทั้งสองคำตอบ

polsk133 16 พฤษภาคม 2014 21:44

#8 ยากมากครับข้อ5 -0- ไม่นึกไม่ฝันว่าจะออกอสมการแนวนี้
ปล.แต่อ่านเฉลยละก้ง่ายจริงๆแหละครับ

passer-by 16 พฤษภาคม 2014 22:00

อ้างอิง:

ข้อความเดิมเขียนโดยคุณ Beatmania (ข้อความที่ 170493)
4. จงหาพหุนาม $P(x)$ ที่มีสัมประสิทธ์เป็นจำนวนเต็มที่

$$P(n)|2557^n+213\times 2014$$

ทุกๆ จำนวนเต็มบวก $n$

สมมติ P nonconstant

ข้อนี้ ถ้าใครรู้ Well known lemma ที่บอกว่า set of prime divisors ของ P(n) มีเป็นอนันต์ ก็จะกลายเป็นข้อง่ายเลยครับ

เลือกจำนวนเฉพาะ p large ที่ (p,2557) =1 และ p| P(a) for some positive integers a

เพราะ p | P(a+p) - P(a) ดังนั้น p | P(a+p)

แสดงว่า $ p | 2557^a +(213)(2014)$ และ $ p | 2557^{a+p} +(213)(2014)$

ดังนั้น $ p | 2557^{a+p} - 2557^a = 2557^a(2557^p-1) \Rightarrow p | 2557^p-1$

จาก FLT p | 2557 -1 = 2556 Contradiction for p large เป็นอนันต์ที่สามารถเลือกได้

ดังนั้น P must be constant polynomial ,say P(x) = 1 or P(x) = -1

Poogunexe 16 พฤษภาคม 2014 22:46

ข้อสามใครมีไอเดียอะไรบ้างอะครับ ไอเดียผมคือ ขั้นแรกพิสูจน์ว่าเดินวนเป็นวงแล้วกลับมาที่เดิมผลรวมระยะทางได้ 0 ต่อไปพิสูจน์ว่าเดิน ขวาแล้วลงกับเดินลงแล้วขวาผลรวมระยะทางเท่ากัน จากนั้นก็ induction ไปเรื่อยๆว่าไม่ว่าจะเดินเละยังไงก็พับให้เป็นทางขวาและลงอย่างเดียวได้ แล้วจะเหลือแค่การเดินจาก (0,N) ไป (M,N) ไป (M,0) ซึ่งผลรวมระยะทางเป็น 0 แต่ผมว่ามันเขียนได้ยากมากเลย ยาวด้วย เห็นเฉลยทีขนลุกเลยครับ

Aquila 16 พฤษภาคม 2014 23:00

ข้อ 8 นี่ทำยังไงอ่ะครับ

ส่วนข้อ 6 มีวิธีที่ไม่ต้องพึ่งสมการ $x^2=y^3+17$ หรือเปล่าครับ

nooonuii 16 พฤษภาคม 2014 23:09

อ้างอิง:

ข้อความเดิมเขียนโดยคุณ polsk133 (ข้อความที่ 170502)
#8 ยากมากครับข้อ5 -0- ไม่นึกไม่ฝันว่าจะออกอสมการแนวนี้
ปล.แต่อ่านเฉลยละก้ง่ายจริงๆแหละครับ

ความแปลกใหม่คือจุดประสงค์แรกที่ผมใส่โจทย์ข้อนี้เข้าไปในคลังข้อสอบครับ และตอนหลังก็ถูกโวตออกมาเป็นข้อสอบจริงๆ

ลืมคิดถึงปัญหาการตีความโจทย์ไปเลย ก็เลยมีคนทำได้น้อยมากๆเมื่อเทียบกับข้อ 1 ซึ่งเราวางไว้ว่าจะเป็นโจทย์ระดับเดียวกัน

แต่ถ้ามองในแง่ดีเด็กไทยเราก็จะได้เรียนรู้อะไรใหม่ๆเพิ่มขึ้นล่ะนะ เวทีใหญ่อย่าง IMO โจทย์ยากกว่านี้เยอะ

ขอให้ผู้ที่ได้ไปต่อทุกคนโชคดีครับ :great:

polsk133 16 พฤษภาคม 2014 23:33

อ้างอิง:

ข้อความเดิมเขียนโดยคุณ Poogunexe (ข้อความที่ 170508)
ข้อสามใครมีไอเดียอะไรบ้างอะครับ ไอเดียผมคือ ขั้นแรกพิสูจน์ว่าเดินวนเป็นวงแล้วกลับมาที่เดิมผลรวมระยะทางได้ 0 ต่อไปพิสูจน์ว่าเดิน ขวาแล้วลงกับเดินลงแล้วขวาผลรวมระยะทางเท่ากัน จากนั้นก็ induction ไปเรื่อยๆว่าไม่ว่าจะเดินเละยังไงก็พับให้เป็นทางขวาและลงอย่างเดียวได้ แล้วจะเหลือแค่การเดินจาก (0,N) ไป (M,N) ไป (M,0) ซึ่งผลรวมระยะทางเป็น 0 แต่ผมว่ามันเขียนได้ยากมากเลย ยาวด้วย เห็นเฉลยทีขนลุกเลยครับ


Beatmania 17 พฤษภาคม 2014 00:19

# 16 วิธี Best Solution (แปลกใหม่และได้คะแนนเต็ม) นะครับ :D

น่านับถือจริงๆ เพราะผมเห็นเฉลยออกมาตั้ง 7 วิธี :blood:

Poogunexe 17 พฤษภาคม 2014 07:29

อ้างอิง:

ข้อความเดิมเขียนโดยคุณ Aquila (ข้อความที่ 170509)
ข้อ 8 นี่ทำยังไงอ่ะครับ

ส่วนข้อ 6 มีวิธีที่ไม่ต้องพึ่งสมการ $x^2=y^3+17$ หรือเปล่าครับ

ข้อแปดผมแสดงก่อนว่ามีชุดบัตร $\frac{n(n-1)}{2} +1$ ใบที่สอดคล้องกับเงื่อนไขที่กำหนด
ประกอบด้วย $1!,2!,3!,...,(n-1)!$ อย่างละ $1,2,3,...,n-1$ ใบ และ $n!$ อีกใบ แล้วอุปนัยว่าการจัดแบบนี้สอดคล้องกับเงื่อนไขทุกๆ n
จากนั้นก็แสดงว่าการสร้าง $n!-1$ ด้วยบัตร $\frac{n(n-1)}{2} $ใบ ทำได้แบบเดียว

polsk133 17 พฤษภาคม 2014 09:13

ข้อ 8..... สอบเสร็จแล้วผมยังไม่คิดจะทำเลย -3-555

polsk133 17 พฤษภาคม 2014 09:33

อ้างอิง:

ข้อความเดิมเขียนโดยคุณ Beatmania (ข้อความที่ 170514)
# 16 วิธี Best Solution (แปลกใหม่และได้คะแนนเต็ม) นะครับ :D

น่านับถือจริงๆ เพราะผมเห็นเฉลยออกมาตั้ง 7 วิธี :blood:

มันเป็นความฟลุคอะครับ คิดมาเปน ชม จู่ๆฟ้าก็ประทานข้อความนั้นมาให้ตอน20นาทีสุดท้าย ไม่แน่ถ้าTMO9 มีรางวัลนี้บางคนก็อาจจะได้นะคับ5555

nooonuii 17 พฤษภาคม 2014 11:26

วิธีของ #16 กรรมการเห็นว่า generalize โจทย์ขึ้นไปอีกครับ

จึงได้ best solution ยินดีด้วยครับ :great:

passer-by 17 พฤษภาคม 2014 12:55

ข้อ 7 เกือบตาย ทด อยู่เกือบ 2 ชั่วโมง :blood: (คาดว่าใครทดข้อนี้ได้ในห้องสอบ อาจไม่เหลือเวลาทดข้ออื่น)

คำตอบคือ E เป็นจุดที่ ทำให้ AD ขนานกับ ฺBE

สมมติ (APD) ตัด (BPE) ที่ Q (คนละจุดกับ P) (Easy to prove by contradiction ว่า 2 วงนี้ไม่สัมผัสกัน และยืนยันได้ว่า Q exists )

อันดับแรก จะ derive สูตร $ O_1O_2$ ให้ได้ก่อนครับ

รายละเอียด ขอไม่เขียนหมดเพราะมันจุกจิกเกิน เอาเป็นว่า วิธีทำ เริ่มจากลากเส้นจาก center ทั้ง 2 จุดมาตั้งฉากกับ DC แล้ว ใช้ Pythaogorus + อัตราส่วน sin ,cos derive ให้อยู่ใน form $ (O_1O_2)^2 = \underbrace{(r_xcos x - r_ycosy)^2+(r_xsin x + r_ysiny)^2}_{(**)} = (r_x)^2+(r_y)^2 - 2r_xr_y\cos D\hat{Q}E $

(จริงๆ ตรงที่ (**) ไว้มันขึ้นกับ ตำแหน่งของ P ด้วยครับว่าให้สามเหลี่ยม 2 รูปนั้นเป็นมุมแหลมหรือมุมป้าน แต่ ตอนจบก็เหมือนกัน โดย r ที่ผมเขียน ก็คือรัศมี ส่วนมุม x,y ก็มาจากมุม DAP , PBE)

จากนั้น ใช้ Law of sine เปลี่ยนค่า r ทั้งสองใน form DQ, QE แล้วมันจะเข้าสูตร law of cosine อย่าง งดงามเป็น $O_1O_2 = \frac{DE}{2\sin Q\hat{P}D}$

คราวนี้ ก็ต้องตามล่าหาจุด E ที่ทำให้ค่า sin ตัวนี้ อิสระจาก P

โดย ถ้า E ที่เป็นจุดที่ผมบอกไป มันจะพิสูจน์ได้ไม่ยากว่า จุด Q, A, B อยู่บนเส้นตรงเดียวกัน

ดังนั้น $\sin Q \hat{P}D = \sin D\hat{A} Q = \sin D\hat{A}B $ ซึ่งไม่ขึ้นกับ P

---------------------------------------------------------------------------------

และเพื่อระบายความอัดอัั้น ที่ทดเกือบ 2 ชั่วโมง ผมมีคำถามคู่ขนานกับข้อนี้ครับ (ใครสนใจก็ลองทดดูได้ครับ ไม่ต้องใช้ตรีโกณ ใช้ความรู้เรขาล้วนๆ)

กำหนดสี่เหลี่ยมนูน ABCD โดย มุม A เป็นมุมป้านและ D เป็นมุมแหลม พิสูจน์ว่ามี จุด E ( $\neq D$) บนเส้นตรง CD ที่สอดคล้องกับ

"ทุก P ($\neq C,D$) บนส่วนของเส้นตรง CD ที่ทำให้ circumcircle (APD) ,(BPE)ตัดกัน อีกจุดที่ Q ($\neq P$) แล้ว set of points Q อยู๋บน fixed circle ไม่ขึ้นกับ P "

Poogunexe 17 พฤษภาคม 2014 22:12

ข้อ 1 น้องเปรมที่ศูนย์ผมเขาทำแบบนี้ครับ
ขั้นแรก ไล่มุมเรื่อยๆจะได้ $\widehat{ACE}=60$
จากนั้นลาก AQ ตัด BE ที่ Q และตัด BC ที่ P ให้สามเหลี่ยม ACQ เป็นสามเหลี่ยมด้านเท่า
ไล่มุมต่อจะได้ $\Delta ABP\sim \Delta ABC$
จากนั้นไล่ด้านไปเรื่อยๆจะได้ $\frac{PQ}{AP} = \frac{BD}{DE} $
ที่นี้พิจารณา $\Delta AEQ$ กับส่วนของเส้นตรง BC โดยทฤษฎีบทเมเนลอส
จะได้ $1=\frac{AB}{BE} \frac{CE}{CQ} \frac{PQ}{AP}=\frac{AB}{BE}\frac{CE}{CQ}\frac{BD}{DE} $
แต่ $AB=CQ$ และ $BE=BC$ ดังนั้นจะได้ $BC\cdot DE=BD\cdot CE$ ตามต้องการ

nooonuii 18 พฤษภาคม 2014 10:50

อ้างอิง:

ข้อความเดิมเขียนโดยคุณ polsk133 (ข้อความที่ 170489)

5. จงหาจำนวนจริง k ที่มากที่สุดที่ทำให้อสมการ $$(k+\frac{a}{b})(k+\frac{b}{c})(k+\frac{c}{a}) \leqslant (\frac{a}{b}+\frac{b}{c}+\frac{c}{a})(\frac{b}{a}+\frac{c}{b}+\frac{a}{c})$$ เป็นจริงทุกจำนวนจริงบวก $a,b,c$

โจทย์ที่แท้จริงที่กรรมการกลางเฉลยไว้คือแบบนี้ครับ

สำหรับ $k\geq 0$ จะได้ว่า
$$
9(k+\frac{a}{b})(k+\frac{b}{c})(k+\frac{c}{a}) \leq (k+1)^3(\frac{a}{b}+\frac{b}{c}+\frac{c}{a})(\frac{b}{a}+\frac{c}{b}+\frac{a}{c})
$$ ทุกจำนวนจริงบวก $a,b,c$

ซึ่งจะได้โจทย์สวยๆอีกหลายข้อเลยเช่น

$$
(2+\frac{a}{b})(2+\frac{b}{c})(2+\frac{c}{a}) \leq 3(\frac{a}{b}+\frac{b}{c}+\frac{c}{a})(\frac{b}{a}+\frac{c}{b}+\frac{a}{c})
$$

$$
(5+\frac{a}{b})(5+\frac{b}{c})(5+\frac{c}{a}) \leq 24(\frac{a}{b}+\frac{b}{c}+\frac{c}{a})(\frac{b}{a}+\frac{c}{b}+\frac{a}{c})
$$

ทุกจำนวนจริงบวก $a,b,c$

ฟินิกซ์เหินฟ้า 18 พฤษภาคม 2014 11:11

1 ไฟล์และเอกสาร
เฉลยข้อที่5 ของสอวน.
ส่วนแรกนะครับ

ฟินิกซ์เหินฟ้า 18 พฤษภาคม 2014 11:13

1 ไฟล์และเอกสาร
ส่วนที่สองนะครับ

ฟินิกซ์เหินฟ้า 18 พฤษภาคม 2014 14:25

1 ไฟล์และเอกสาร
ฉบับภาษาอังกฤษนะครับ

ฟินิกซ์เหินฟ้า 18 พฤษภาคม 2014 14:26

1 ไฟล์และเอกสาร
ข้อ 3-4 นะครับ

ฟินิกซ์เหินฟ้า 18 พฤษภาคม 2014 14:33

1 ไฟล์และเอกสาร
ข้อ 5-6 ของวันที่สอง นะครับ

ฟินิกซ์เหินฟ้า 18 พฤษภาคม 2014 14:34

1 ไฟล์และเอกสาร
สุดท้ายครับ
ข้อ 7-8

nooonuii 20 พฤษภาคม 2014 12:57

อ้างอิง:

ข้อความเดิมเขียนโดยคุณ polsk133 (ข้อความที่ 170489)
5. จงหาจำนวนจริง $k$ ที่มากที่สุดที่ทำให้อสมการ $$(k+\frac{a}{b})(k+\frac{b}{c})(k+\frac{c}{a}) \leqslant (\frac{a}{b}+\frac{b}{c}+\frac{c}{a})(\frac{b}{a}+\frac{c}{b}+\frac{a}{c})$$ เป็นจริงทุกจำนวนจริงบวก $a,b,c$

โจทย์คู่ขนานของข้อ $5$ แต่ง่ายกว่า

จงหาจำนวนจริง $k$ ที่น้อยที่สุดที่ทำให้อสมการ

$$
(k+\frac{a}{b})(k+\frac{b}{c})(k+\frac{c}{a}) \geqslant (a+b+c)(\frac{1}{a}+\frac{1}{b}+\frac{1}{c})
$$
เป็นจริงทุกจำนวนจริงบวก $a,b,c$

oyyks 21 พฤษภาคม 2014 05:05

อ้างอิง:

ข้อความเดิมเขียนโดยคุณ nooonuii (ข้อความที่ 170733)
โจทย์คู่ขนานของข้อ $5$ แต่ง่ายกว่า

จงหาจำนวนจริง $k$ ที่น้อยที่สุดที่ทำให้อสมการ

$$
(k+\frac{a}{b})(k+\frac{b}{c})(k+\frac{c}{a}) \geqslant (a+b+c)(\frac{1}{a}+\frac{1}{b}+\frac{1}{c})
$$
เป็นจริงทุกจำนวนจริงบวก $a,b,c$

แทน a=b=c=1 เหมือนเดิม ได้ $(k+1)^3 \ge 9$
ให้ $(k+1)^3 = 9$ กระจายทั้งสองข้างออกมาแล้ว am-gm ที่เหลือได้เลยครับ

ฟินิกซ์เหินฟ้า 21 พฤษภาคม 2014 21:09

อ้างอิง:

ข้อความเดิมเขียนโดยคุณ nooonuii (ข้อความที่ 170733)
โจทย์คู่ขนานของข้อ $5$ แต่ง่ายกว่า

จงหาจำนวนจริง $k$ ที่น้อยที่สุดที่ทำให้อสมการ

$$
(k+\frac{a}{b})(k+\frac{b}{c})(k+\frac{c}{a}) \geqslant (a+b+c)(\frac{1}{a}+\frac{1}{b}+\frac{1}{c})
$$
เป็นจริงทุกจำนวนจริงบวก $a,b,c$

แทน $a=b=c$ จะไดัว่า $k \ge \sqrt[3]{9} -1$
ให้ $A=\dfrac{a}{b}+\dfrac{b}{c}+\dfrac{c}{a},B=\dfrac{a}{c}+\dfrac{b}{a}+\dfrac{c}{b}$
จะแสดงว่า $k=\sqrt[3]{9} -1$ ทำให้อสมการเป็นจริง
$k^3+Ak^2+Bk+1 \ge A+B+3$
จาก $(k+1)^3=9$
$\Leftrightarrow A+B+3+(3-A)k^2+(3-B)k \le 9$
$\Leftrightarrow A(k^2-1)+B(k-1) \ge 3k^2+3k-6 $
เนื่องจาก $k=\sqrt[3]{9} -1 > 1$
ดังนั้น $A(k+1)+B \ge 3(k+2)$
หรือ $(A-3)k+(A-3)+(B-3) \ge 0$
ซึ่งเป็นจริงโดยอสมการ $AM-GM$
ดังนั้นค่าน้อยที่สุดของจำนวนจริง $k$ คือ $\sqrt[3]{9} -1$

Panithi Vanasirikul 21 พฤษภาคม 2014 21:43

มีพี่คนไหนที่ได้ไปเเข่งต่างประเทศมั้ยครับ

ACFEGIN 23 พฤษภาคม 2014 20:47

ข้อสอบปีนี้แต่ละข้อโหดจริงครับ

ฟินิกซ์เหินฟ้า 23 พฤษภาคม 2014 21:40

อ้างอิง:

ข้อความเดิมเขียนโดยคุณ Poogunexe (ข้อความที่ 170492)
2. จงหาฟังก์ชัน $ f:\mathbb{R} \rightarrow \mathbb{R} $ ทั้งหมดที่สอดคล้องกับเงื่อนไข
$ f(xy-1)+f(x)f(y)=2xy-1 $
สำหรับทุกจำนวนจริง $x$ และ $y$

เพิ่มเติมอีกวิธี(ในเฉลย)นะครับ
แทน $y =0$ จะได้ $ f(-1) +f(x)f(0) = -1 $
แต่ constant function ไม่ใช่คำตอบ ดังนั้น $f(0) = 0$
แทน $x=y=1$ ทำให้ได้ว่า $f(1) = 1,-1$
แทนค่า $y= 1$ ในสมการเริ่มต้นจะได้ $f(x-1) + f(x)f(1) = 2x-1.......(1)$
แบ่ง case $f(1)=1$ และ $f(1)=-1$
แทน $x$ ด้วย $xy$ ใน $(1)$
จะได้ $f(xy-1)+f(xy)f(1)=2xy-1$ (ตรงนี้ครับที่สวยมากๆ)
ถ้า $f(1)=1$ ไดัว่า $f(xy)=f(x)f(y)$
แล้วใช้ $(1)$ แก้หา $f(x)$ ออกมาจะได้ $f(x)=x$
ถ้า $f(1)=-1$ ทำในทำนองเดียวกัน
จะได้ $f(x)=-x^2$ ครับ

Panithi Vanasirikul 23 พฤษภาคม 2014 22:39

ที่เขียนกันนี่ม.ไรครับ เริ่มงงเองเเล้ว

Poogunexe 23 พฤษภาคม 2014 22:52

อ้างอิง:

ข้อความเดิมเขียนโดยคุณ Panithi Vanasirikul (ข้อความที่ 171016)
ที่เขียนกันนี่ม.ไรครับ เริ่มงงเองเเล้ว

สอบไก้ตั้งแต่ ม.1-ม.5 ครับ

Panithi Vanasirikul 23 พฤษภาคม 2014 23:03

ผมอยากเพิ่มโจทย์ อ่ะสักข้อละกัน ให้ a,b เป็นสมาชิก R+ ที่สอดคล้องกับ $a+b=ab=a^2-b^2$ จงหาว่า จำนวนเต็มที่น้อยสุดที่มากกว่าa+ จำนวนเต็มที่น้อยที่สุดที่มากกว่าb เท่ากับเท่าไหร่ สอวน. 56 จ้า

นกกะเต็นปักหลัก 24 พฤษภาคม 2014 08:12

$a^2-b^2=(a+b)(a-b)=a+b$
แต่ a+b >0
ได้ว่า a-b=1
$2b+1=b^2+b$
$b^2-b-1=0$
b=$\frac{1+\sqrt{5} }{2}$
และa=b+1=$\frac{3+\sqrt{5} }{2}$


เวลาที่แสดงทั้งหมด เป็นเวลาที่ประเทศไทย (GMT +7) ขณะนี้เป็นเวลา 02:15

Powered by vBulletin® Copyright ©2000 - 2024, Jelsoft Enterprises Ltd.
Modified by Jetsada Karnpracha